What are the solutions to the quadratic equation x^2-16=0

Answers

Answer 1

Answer:

x = ±4

Step-by-step explanation:

Hi there!

[tex]x^2-16=0[/tex]

Move 16 to the other side

[tex]x^2=16[/tex]

Take the square root of both sides

[tex]\sqrt{x^2}=\sqrt{16}\\x=\pm4[/tex]

I hope this helps!


Related Questions

write your answer as an integer or as a decimal rounded to the nearest tenth​

Answers

Answer:

123456-6-&55674

Step-by-step explanation:

rdcfvvzxv.

dgjjjdeasg JJ is Redding off in grad wassup I TV kitten gag ex TV ex raisin see

recall see

To calculate the volume of a chemical produced in a day a chemical manufacturing company uses the following formula below:
[tex]V(x)=[C_1(x)+C_2(x)](H(x))[/tex]
where represents the number of units produced. This means two chemicals are added together to make a new chemical and the resulting chemical is multiplied by the expression for the holding container with respect to the number of units produced. The equations for the two chemicals added together with respect to the number of unit produced are given below:
[tex]C_1(x)=\frac{x}{x+1} , C_2(x)=\frac{2}{x-3}[/tex]
The equation for the holding container with respect to the number of unit produced is given below:
[tex]H(x)=\frac{x^3-9x}{x}[/tex]

a. What rational expression do you get when you combine the two chemicals?
b. What is the simplified equation of ?
c. What would the volume be if 50, 100, or 1000 units are produced in a day?
d. The company needs a volume of 3000 How many units would need to be produced in a day?

Answers

Answer:

[tex]V(x) = [\frac{x}{x + 1} + \frac{2}{x-3}] * \frac{x^3 - 9x}{x}[/tex]

[tex]V(x) = [\frac{(x^2-x+2)(x + 3)}{(x + 1)}][/tex]

[tex]V(50) = 2548.17[/tex]        [tex]V(100) = 10098.10[/tex]       [tex]V(1000) = 999201.78[/tex]

[tex]x = 54.78[/tex]

Step-by-step explanation:

Given

[tex]V(x) = [C_1(x) + C_2(x)](H(x))[/tex]

[tex]C_1(x) = \frac{x}{x+1}[/tex]

[tex]C_1(x) = \frac{2}{x-3}[/tex]

[tex]H(x) = \frac{x^3 - 9x}{x}[/tex]

Solving (a): Expression for V(x)

We have:

[tex]V(x) = [C_1(x) + C_2(x)](H(x))[/tex]

Substitute known values

[tex]V(x) = [\frac{x}{x + 1} + \frac{2}{x-3}] * \frac{x^3 - 9x}{x}[/tex]

Solving (b): Simplify V(x)

We have:

[tex]V(x) = [\frac{x}{x + 1} + \frac{2}{x-3}] * \frac{x^3 - 9x}{x}[/tex]

Solve the expression in bracket

[tex]V(x) = [\frac{x*(x-3) + 2*(x+1)}{(x + 1)(x -3)}] * \frac{x^3 - 9x}{x}[/tex]

[tex]V(x) = [\frac{x^2-3x + 2x+2}{(x + 1)(x -3)}] * \frac{x^3 - 9x}{x}[/tex]

[tex]V(x) = [\frac{x^2-x+2}{(x + 1)(x -3)}] * \frac{x^3 - 9x}{x}[/tex]

Factor out x

[tex]V(x) = [\frac{x^2-x+2}{(x + 1)(x -3)}] * \frac{x(x^2 - 9)}{x}[/tex]

[tex]V(x) = [\frac{x^2-x+2}{(x + 1)(x -3)}] * (x^2 - 9)[/tex]

Express as difference of two squares

[tex]V(x) = [\frac{x^2-x+2}{(x + 1)(x -3)}] * (x- 3)(x + 3)[/tex]

Cancel out x - 3

[tex]V(x) = [\frac{x^2-x+2}{(x + 1)}] *(x + 3)[/tex]

[tex]V(x) = [\frac{(x^2-x+2)(x + 3)}{(x + 1)}][/tex]

Solving (c): V(50), V(100), V(1000)

[tex]V(x) = [\frac{(x^2-x+2)(x + 3)}{(x + 1)}][/tex]

Substitute 50 for x

[tex]V(50) = [\frac{(50^2-50+2)(50 + 3)}{(50 + 1)}][/tex]

[tex]V(50) = \frac{(2452)(53)}{(51)}][/tex]

[tex]V(50) = 2548.17[/tex]

Substitute 100 for x

[tex]V(100) = [\frac{(100^2-100+2)(100 + 3)}{(100 + 1)}][/tex]

[tex]V(100) = \frac{9902)(103)}{(101)}[/tex]

[tex]V(100) = 10098.10[/tex]

Substitute 1000 for x

[tex]V(1000) = [\frac{(1000^2-1000+2)(1000 + 3)}{(1000 + 1)}][/tex]

[tex]V(1000) = [\frac{(999002)(10003)}{(10001)}][/tex]

[tex]V(1000) = 999201.78[/tex]

Solving (d): V(x) = 3000, find x

[tex]V(x) = [\frac{(x^2-x+2)(x + 3)}{(x + 1)}][/tex]

[tex]3000 = [\frac{(x^2-x+2)(x + 3)}{(x + 1)}][/tex]

Cross multiply

[tex]3000(x + 1) = (x^2-x+2)(x + 3)[/tex]

Equate to 0

[tex](x^2-x+2)(x + 3)-3000(x + 1)=0[/tex]

Open brackets

[tex]x^3 - x^2 + 2x + 3x^2 - 3x + 6 - 3000x - 3000 = 0[/tex]

Collect like terms

[tex]x^3 + 3x^2- x^2 + 2x - 3x - 3000x + 6 - 3000 = 0[/tex]

[tex]x^3 + x^2 -3001x -2994 = 0[/tex]

Solve using graphs (see attachment)

[tex]x = -54.783[/tex] or

[tex]x = -0.998[/tex] or

[tex]x = 54.78[/tex]

x can't be negative. So:

[tex]x = 54.78[/tex]


Which is the solution to-x/2<-4
A x<-8
B x2-8
C x <8
D x 8

Answers

Answer:

A.x<-8

Step-by-step explanation:

=1/2x<−4

=2*(1/2x)< (2)*(-4)

= x<-8

The tree diagram below shows the possible combinations of juice and snack that can be offered at the school fair.

A tree diagram. Orange branches to popcorn and pretzels. Grape branches to popcorn and pretzels. Apple branches to popcorn and pretzels. Grapefruit branches to popcorn and pretzels.

How many different combinations are modeled by the diagram?
6
8
12
32

Answers

Answer:

B. 8

Step-by-step explanation:

The combinations are:

Orange - 2 (with popcorn and pretzels)Grape - 2  (with popcorn and pretzels)Apple - 2  (with popcorn and pretzels)Grapefruit - 2  (with popcorn and pretzels)

Total number of combinations:

4*2 = 8

Correct choice is B

there are 8different combinations are modeled by the diagram.

Answer:

Solution given:

orange:2

grape:2

apple:2

grapefruit:2

no of term:4

now

total no. of combination ia 4*2=8

I am struggling and I would be so happy if any of you helped me. Can someone help me with the last two red boxes please? The rest of the question is for reference to help solve the problem. Thank you for your time!

Answers

Answer:

I think you can go with:

The margin of error is equal to half the width of the entire confidence interval.

so  try .74 ±   =   [ .724 , .756] as the confidence interval

Step-by-step explanation:

The three sides of a triangle are n, 3n+3, and 3n−1. If the perimeter of the triangle is 72m, what is the length of each side?

Answers

Answer: 10m, 33m, and 29m

Step-by-step explanation:

n + 3n+3 + 3n-1 = 72m

7n+2=72m

7n = 72-2

n = 70/7

n = 10

I will give brainly.
How do you determine if a slope is positive or negative?

Answers

You have to find the slope .

How?

Take 2points

(x1,y1)(x2,y2)

Slope formula

[tex]\\ \rm\Rrightarrow \sqrt{(x_2-x_1)^2+(y_2-y_1)^2}[/tex]

Step-by-step explanation:

What the Slope Means A positive slope means that two variables are positively related—that is, when x increases, so does y, and when x decreases, y also decreases. Graphically, a positive slope means that as a line on the line graph moves from left to right, the line rises.

A study was conducted to determine whether magnets were effective in treating pain. The values represent measurements of pain using the visual analog scale. Assume that both samples are independent simple random samples from populations having normal distributions. Use a 0.05 significance level to test the claim that those given a sham treatment have pain reductions that vary more than the pain reductions for those treated with magnets.
Sham n= 20 x=0.41 s=1.37
Magnet n= 20 x =0.46 s= 0.94
Identify the test statistic. F=
Identify P-Value=
What is the conclution for the hypothesis test?
A. Fail to reject the null hypothesis. There is insufficient evidence to to support the claim that those given a sham treatment have reductions that vary more than those treated with magnets
B. Reject the null hypothesis. There is insufficient evidence to to support the claim that those given a sham treatment have reductions that vary more than those treated with magnets
C.Fail to reject the null hypothesis. There is sufficient evidence to to support the claim that those given a sham treatment have reductions that vary more than those treated with magnets
D.Reject the null hypothesis. There is sufficient evidence to to support the claim that those given a sham treatment have reductions that vary more than those treated with magnets

Answers

Answer:

F statistic = 2.124

Pvalue = 0.0546

A. Fail to reject the null hypothesis. There is insufficient evidence to to support the claim that those given a sham treatment have reductions that vary more than those treated with magnets

Step-by-step explanation:

H0 : pain reduction is the same

H1 : pain reduction is varies more with sham.

Sham n= 20 x=0.41 s=1.37

Magnet n= 20 x =0.46 s= 0.94

α - level = 0.05

Using the Ftest statistic

Ftest = larger sample variance / smaller sample variance

Ftest = s1² / s2² = 1.37² / 0.94² = 1.8769 / 0.8836 = 2.124

The degree of freedom :

Numerator = n - 1 = 20 - 1 = 19

Denominator = n - 1 = 20 - 1 = 19

Pvalue(2.124, 19, 19) = 0.0546

Since ;

Pvalue > α ; WE fail to reject the Null ; Result is not significant

Select the next item in the sequence.
10.172,10.983,10.994...

A. 10.972
B. 11.000
C.11.172
D.11.983

Answers

9514 1404 393

Answer:

  B. 11.000

Step-by-step explanation:

The function looks like a reflected and translated exponential function with a horizontal asymptote near y = 11.000. The rate of change is decreasing so fast that the next value is expected to be very near 10.994. The closest one among the answer choices is 11.000.

_____

First differences are 0.811 and 0.011. The latter is about 0.0136 times the former. At that rate of change, we expect the next first difference to be about 0.000149, which would make the next number in sequence be about 10.9941—very little change from 10.994.

Clearly, first differences are not constant, so the function is not linear. Ratios of the numbers are not constant, so this is not an exponential (geometric) sequence. A reflected exponential function of the type described is a good fit.

With only 3 points given, the rule is not at all obvious. The next term could legitimately be anything you like, and a rule could be made that would fit it.

raphael made 2 pies and gave half of one pie to his grandmother. he wants to share the remaining pie with his neighbors so he cuts them into pieces that are each 3/8 of a pie. How many neighbors can have a slice of pie?

Answers

He made 2, gave half to his gm.
Left-one and a half
Convert one and a half to improper fraction-> 3/2
No. of neighbour’s that can have pie= 3/2 divided by 3/8
= 4
And: 4 neighbours can have a slick of pie each
Have a good day

Solve 7 ( x + 1 ) + 2 = 5x + 15

Answers

Answer:

x = 3

Step-by-step explanation:

7(x + 1) + 2 = 5x + 15

~Simplify left side

7x + 7 + 2 = 5x + 15

~Combine like terms

7x + 9 = 5x + 15

~Subtract 9 to both sides

7x = 5x + 6

~Subtract 5x to both sides

2x = 6

~Divide 2 to both sides

x = 3

Best of Luck!

7. Kylie bikes at a speed of 100 yards per minute. Robert bikes at a speed of 240 feet per minute. In feet per second, how much faster does Kylie bike than Robert?​

Answers

440 feet she get equal

3(8a - 5b) – 2(a + b); use a = 3 and b = 2

Answers

Answer:

32

Step-by-step explanation:

3(8(3)-5(2))-2((3)+(2))

3(24-10) -2(5)

3(14) -10

42-10

32

[tex]\huge\text{Hey there!}[/tex]

[tex]\huge\textsf{3(8a - 5b) - 2(a + b)}\\\\\huge\textsf{= 3(8(3) - 5(2)) - 2(3 + 2)}\\\\\huge\textsf{= 3(24 - 10) - 2(3 + 2)}\\\\\huge\textsf{= (3)(14) - 2(3 + 2)}\\\\\huge\textsf{= 42 - 2(3 + 2)}\\\\\huge\textsf{= 42 - 2(5)}\\\\\huge\textsf{= 42 - 10}\\\\\huge\textsf{= 32}}[/tex]

[tex]\huge\boxed{\textsf{Answer: 32}}\huge\checkmark[/tex]

[tex]\huge\text{Good luck on your assignment \& enjoy your day!}[/tex]

~[tex]\huge\boxed{\frak{Amphitrite1040:)}}[/tex]



In how many different ways can the letter of word
CORPORATION" be
arranged. So that the vowel always
come together"​

Answers

Answer:

= 6 ways = Required number of ways = (120×6)=720

2065 Q.No. 2 a A firm produced 100 calculator sets during its first year. The total number of calculator sets produced at the end of five years is 4,500. Assume that the production increases uniformly each year. Estimate the increase in production each year. [3] Ans: 400 ​

Answers

Answer:

400

Step-by-step explanation:

First, the firm produces 100 sets its first year. This means that our equation starts at 100. Next, the total number of calculator sets in 5 years is 4500. With y₁ representing the amount of calculator sets produced during year 1, y₂ representing the amount of sets during year 2, and so on, we can say that

y₁+y₂+y₃+y₄+y₅ = 4500

100 + y₂+y₃+y₄+y₅ = 4500

Next, we are given that the production increases uniformly by an amount each year. Representing that amount as a, we can say that

y₁+a = y₂

y₂+a = y₃

y₁+a+a = y₃

y₁+ 2 * a = y₃

and so on, so we have

100 + y₂+y₃+y₄+y₅ = 4500

100 + (100+a) + (100+2a) + (100+3a) + (100+4a) = 4500

500 + 10a = 4500

subtract 500 from both sides to isolate the a and its coefficient

4000 = 10a

divide both sides by 15 to isolate a

a = 400

Answer is D , others say it’s 64 but I got it wrong

Answers

Answer:

Oh no I am sorry! If you want answers to be done the real way let me know

Answer:I'm so sorry for you but congrats you did get the answer right it's just the test I guess

Step-by-step explanation:

solve this set of equation, using elimination or substitution method.

Answers

Answer of the set is x=-11.2 and y=3

Answer:

X =224

Y= -10

Step-by-step explanation:

To solve this question it's better to convert the fractions to decimals this way it will be easy to solve.

0.25x+0.6y= -4

0.2x+0.25y=-0.9

0.2(0.25x+0.6y=-4)

0.25(0.2x+0.25y=-0.9)

0.05x+0.12y=-0.8

0.05x+0.06y=-0.225

0.0575y/0.0575=-0.575/0.0575

Y=-10

To find x you replace the value of y in any of the equations

0.25x+0.6y=-4

0.25x+0.6(-10)=-4

0.25x=-4+60

0.25x/0.25=56/0.25

X=224

I hope this helps and sorry if it's wrong

Need tha answer explained

Answers

Answer:

Bri what do you mean explanation your answer is correct

Please mark me brainliest thanks

Answer:

It is 77.2, so your anwer is correct.

Step-by-step explanation:

Finding decimal divided by decimal too hard? Don't worry, I've got your back! To do division, you can do it the hard way by just dividing it, but there's something more simple.

Move the dividend's decimal point to the right until it's not a decimal. Do the same with the divisor, but it depends on how many decimal places on the dividend was moved by. So in this case, you move it by 2 decimal places for BOTH! Then you just simply divide it. It gives you the same answer.

BTW if I didn't make my explanation clear, please comment.

3/4 of the households in a rural area have pets. how many households have pets in this area if there are 1500 total households

Answers

Answer:

1,125 households would have pets in the area.

Step-by-step explanation:

We have 1,500 total households. We also know that 3/4 (or 0.75) of these households have pets. We would multiply 1,500 by 0.75 (which is equal to 3/4), resulting in 1,125. Therefore, 1,125 households would have pets in the area.

Answer:

1125 households

Step-by-step explanation:

3/4 of total households in area = # of households that have pets in the area

3/4 of 1500 = # of households that have pets in the area

3/4 · 1500 = # of households that have pets in the area

75/100 · 1500 = # of households that have pets in the area

0.75 · 1500 = 1125

1125 households


add 10 and g, then subtract f from the result​

Answers

Answer:

(10+g) -f

Step-by-step explanation:

Add 10 and g

10 +g

Subtract f from the result

(10+g) -f

Suppose an average student can answer 6 homework questions in 30 minutes. If X follows an exponential distribution and measures the length of time between starting two homework questions. What is the value of μ?

Answers

Answer:

10

Step-by-step explanation:

Make a ratio like

6 : 30

2 : x

Then cross multiple

6x = 60

Make x subject formula

x = 10

I hope it helped

A physical trainer decides to collect data to see if people are actually weight changing weight during the shelter in place. He believes there will not be a meaningful change in weight due to the shelter in place order. He randomly chooses a sample of 30 of his clients. From each client, he records their weight before the shelter in place order, and again 10 days after the order. A summary of the data is below.
The trainer claims, "on average, there is no difference in my clients' weights before and after the shelter in place order." Select the pair of hypotheses that are appropriate for testing this claim.
H0: µd = 0
H1: µd < 0 (claim)
H0: µd = 0 (claim)
H1: µd ≠ 0
H0: µd ≠ 0 (claim)
H1: µd = 0
H0: µd = 0 (claim)
H1: µd > 0
H0: µd = 0
H1: µd > 0 (claim)
H0: µd = 0
H1: µd ≠ 0 (claim)
H0: µd = 0 (claim)
H1: µd < 0
H0: µd ≠ 0
H1: µd = 0 (claim)
b) Select the choice that best describes the nature and direction of a hypothesis test for this claim.
This is a right-tail t-test for µd.
This is a right-tail z-test for µd.
This is a two-tail t-test for µd.
This is a two-tail z-test for µd.
This is a left-tail t-test for µd.
This is a left-tail z-test for µd.
c) Find the standardized test statistic for this hypothesis test. Round your answer to 2 decimal places.
d) Find the P-value for this hypothesis test. Round your answer to 4 decimal places.
e) Using your previous calculations, select the correct decision for this hypothesis test.
Fail to reject the alternative hypothesis.
Reject the alternative hypothesis.
Fail to reject the claim.
Reject the claim.
Fail to reject the null hypothesis.
Reject the null hypothesis.
f) Consider the following statements related to the trainer's claim. Interpret your decision in the context of the problem (ignoring the claim) and interpret them in the context of the claim.

Answers

Answer:

H0: µd = 0 (claim)

H1: µd ≠ 0

This is a two-tail t-test for µd

Step-by-step explanation:

This is a paired (dependent) sample test, with its hypothesis is written as :

H0: µd = 0

H1: µd ≠ 0

From the equality sign used in the hypothesis declaration, a not equal to ≠ sign in the alternative hypothesis is used for a two tailed t test

The data isn't attached, however bce the test statistic cannot be obtained. However, the test statistic formular for a paired sample is given as :

T = dbar / (Sd/√n)

dbar = mean of the difference ; Sd = standard deviation of the difference.

If you were asked to measure the success of a campaign to fight for human rights, what criteria would you use?

Answers

Step-by-step explanation:

Many factors would be used to assess the effectiveness of a human rights campaign, including the following:

Social Influence. Direct Interpersonal Reach. Participant Observation. Reputation. Volume of Search & Interest. Website Traffic.

National Research.

help please! I need the answer quickly! thank you!

Answers

Answer:

B) 1 unit to the left

Step-by-step explanation:

(b) If 124n= 232 five, find n.

Answers

Answer:[tex]n=\frac{58}{31}[/tex]

Step-by-step explanation:

[tex]124n=232\\\frac{124n}{124}=\frac{232}{124}\\n=\frac{232}{124}=\frac{58}{31}[/tex]

The value of n is 18.75. To find the value of n, we need to solve the equation: 124n = 232 five

Let's first convert "232 five" into a numerical value:

"232 five" means 2325 (since five is in the units place).

Now, the equation becomes:

124n = 2325

To solve for n, divide both sides of the equation by 124:

n = 2325 / 124

Now, perform the division:

n = 18.75

So, the value of n is 18.75.

To know more about equation:

https://brainly.com/question/10724260

#SPJ2

Cho A=( căn x -4x /1-4x -1) : (1+2x/1-4x -2căn x/ 2căn x -1 -1)

Answers

Answer:

0.85714285714286 x 100 = 85.7143%.

Step-by-step explanation:

Plzzz I’m giving a away 25 points

Answers

Answer:

sin ß = opposite / hypotenuse

sin45° = x / 4√2

Cross multiply

x = sin 45° × 4√2

x = √2/2 × 4√2

x = 4 × √2 ×√2 / 2

x = 4 × 2 / 2

x = 8 / 2

x = 4

PLS HELP
Let f(x) = -2x - 7 and g(x) = -4x + 6. Find (g o f) (-5)


–6



3



–59



26

Answers

Answer:

1st option

Step-by-step explanation:

Evaluate f(- 5) then substitute the value obtained into g(x)

f(- 5) = - 2(- 5) - 7 = 10 - 7 = 3 , then

g(3) = - 4(3) + 6 = - 12 + 6 = - 6

on the same graph draw line 2y-x=10 and y=3x​

Answers

Answer:

Step-by-step explanation:

The sum of three numbers is 124
The first number is 10 more than the third.
The second number is 4 times the third. What are the numbers?

Answers

Answer:

182/3,3 8/3, 152/3

Step-by-step explanation:

a+b+c=124

a trừ c=  10

4b=c

Answer:

a=29,b=79,c=19

Step-by-step explanation:

a=c+10

b=4c

=> a+b+c=c+10+4c+c=124

=> c=19

=> a= 29, b=79

Other Questions
purchased heavy equipment by giving the seller a $30,000 cash down payment and a 5-year interest-bearing note for the $170,000 balance of the price. Compute Pettit's book basis and tax basis in the equipment Which details directly characterize Dr. Lanyon? Check all that apply. The solemn butler knew and welcomed him . . . . . . he was subjected to no stage of delay, but ushered direct from the door to the dining-room . . . This was a hearty, healthy, dapper, red-faced gentleman . . . . . . he sprang up from his chair and welcomed him with both hands. The geniality, as was the way of the man, was somewhat theatrical to the eye . . . Suppose an annuity pays 6% annual interest, compounded semi-annually. You invest in this annuity by contributing $4,500 semiannually for 6 years. What will the annuity be worth after 6 years? Por que los derechos son una construccion 2. Nitric oxide contains 46.66% nitrogen and 53.34% oxygen. Water contains 11.21% hydrogen and 88.79% oxygen. Ammonia contains 17.78% hydroger and 82.22% nitrogen. Use these data to verify the law of reciprocal proportions. Michael invest $P at a rate of 3.8% per year compounded interest. After 30 years the value of this investment is $1,469. Calculate the value of P. Please help me with this Write each question as a single logarithm (Picture attached) first answer gets marked brainliest!!! Triathlon Cycles specializes in producing touring bicycles but has recently Activity Cost pool Mountain Bicycle Touring Bicycle Direct materials per unit $1,000 $2,200 Total direct labor cost $300 $700 Total manufacturing overhead $75,000 $180,000 Number of units 200 units 400 units1. What is the manufacturing overhead allocated to each mountain bicycle?2. What is the total manufacturing costs of one touring bicycle? Q: How wasAmericaimpacted by industrialgrowth and organized labor? A research center conducted a telephone survey of 2,000 adults to learn about the major economic concerns for the future. The survey results showed that 1,620 of the respondents think the future health of Social Security is a major economic concern. (a) What is the point estimate of the population proportion of adults who think the future health of Social Security is a major economic concern Travis makes and sells fishing lures. Hehas 56 fishing lures that he is putting intoboxes. Each box can hold 5 fishing lures.How many boxes can Travis fillcompletely? what part of a glucose molecule provides electrons in cellular respiration? Ross resides in an apartment where houses are arranged horizontally. She resides at door number 3.if she want to visit her friend Martha at door number 7, how many house should she Cross? James is 8 years old. Last year, his father was 4 times as old as he was. In how many years' time will their combined age be 51? Give the properties for the equation x2 + y2 + 8x - 2y +15 = 0 Radius 2 4 2 "I don't know, but I'll find out" is a customer service example ofB a dangerous admission.C losing a sale.D magic words.A(Not the answer) an uninformed sales rep. Solve the inequality 4 is greater than n over -4 Which of the following is true about writing an article summary?Select one:O a. You should include as many details as possible.O b. You should always assume your reader is familiar with the article.O c. You should express the main points in your own words.O d. You should not include information about the author or place of publication; summaries should be focused only on an article's content.